LSAT and Law School Admissions Forum

Get expert LSAT preparation and law school admissions advice from PowerScore Test Preparation.

 Administrator
PowerScore Staff
  • PowerScore Staff
  • Posts: 8917
  • Joined: Feb 02, 2011
|
#41631
Complete Question Explanation
(The complete setup for this game can be found here: lsat/viewtopic.php?t=2813)

The correct answer choice is (B)

Global, Cannot be true questions such as this one can be time consuming, especially since the correct answer choice is difficult to prephrase using Not Laws.

Essentially, this question asks us to determine which two presentations cannot be the ones given by J. Although J is not directly affected by the first rule, recall that all six variables are affected by the Separation Principle involving K, directly or not. This is where templates prove particularly advantageous, as they can be used to quickly identify what could be true, and what cannot be.

If your approach does not involve templates, make sure to refer regularly to the Local setups you have already made. Such setups can prove which two presentations could be given by J, thereby eliminating incorrect answer choices without much additional work.

Answer choice (A) is incorrect, because J could give the first and third presentations (Template 4: see setup discussion). You can also eliminate this answer choice by referring to the local setup for Question #8.

Answer choice (B) is the correct answer choice, because if J gave the first and fourth presentations, K would be unable to give her three presentations in accordance with the first rule. Since none of the templates allow for J to give her presentations in the manner specified here, we can safely conclude that answer choice (B) cannot be true, and is therefore correct.

Answer choice (C) is incorrect, because J could give the first and fifth presentations (Template 4). You can also eliminate this answer choice by referring to the local setup for Question #8.

Answer choice (D) is incorrect, because J could give the second and third presentations (Template 3).

Answer choice (E) is incorrect, because J could give the second and fourth presentations (Template 2).

Get the most out of your LSAT Prep Plus subscription.

Analyze and track your performance with our Testing and Analytics Package.